Find The Value Of The Floor Function

Algebra Level 5

Find 33 29 × 41 37 × 49 45 × × 2009 2005 × 2017 2013 . \left\lfloor \frac{33}{29}\times \frac{41}{37}\times \frac{49}{45}\times \cdots \times \frac{2009}{2005}\times \frac{2017}{2013} \right\rfloor .

Details and assumptions

The function x : R Z \lfloor x \rfloor: \mathbb{R} \rightarrow \mathbb{Z} refers to the greatest integer smaller than or equal to x x . For example 2.3 = 2 \lfloor 2.3 \rfloor = 2 and 5 = 5 \lfloor -5 \rfloor = -5 .


The answer is 8.

This section requires Javascript.
You are seeing this because something didn't load right. We suggest you, (a) try refreshing the page, (b) enabling javascript if it is disabled on your browser and, finally, (c) loading the non-javascript version of this page . We're sorry about the hassle.

5 solutions

In this solution, we use telescoping series & inequalities to approach the problem. The motivation is that: we cannot calculate the fraction exactly at hand, & that is not asked for. We can approximate it, at best, to get an exact floored value. We will make the required value less than and greater than 2 fairly close values, which are easy to calculate, since they telescope. This is fairly a standard technique in such a problem involving a fraction whose numerators & denominators are in A.P.

Note that I = 33 29 41 37 49 45 2009 2005 2017 2013 I = \frac{33}{29} \cdot \frac{41}{37} \cdot \frac{49}{45} \cdots \frac{2009}{2005} \cdot \frac{2017}{2013}

< 29 25 37 33 2013 2007 < \frac{29}{25} \cdot \frac{37}{33} \cdot \cdots \frac{2013}{2007} .

Multiplying I I to both sides we see that the right side telescopes to 2017 25 \frac{2017}{25} , thereby I < 2017 25 I < \sqrt{\frac{2017}{25}} .

Similarly, I > 37 33 45 41 2013 2009 2025 2017 I 2 > 2025 29 I > 2025 29 I > \frac{37}{33} \cdot \frac{45}{41} \cdots \frac{2013}{2009} \cdot \frac{2025}{2017} \Rightarrow I^2 > \frac{2025}{29} \Rightarrow I > \sqrt{\frac{2025}{29}} .

Using the above estimates, we see: I = 8 \lfloor I \rfloor = \boxed{8} .

nice one singh

Amritpal Singh - 7 years, 4 months ago

a good one

Malay Pandey - 7 years, 3 months ago
Santanu Banerjee
Jan 3, 2014

Let A = 33 29 × 41 37 × × 2017 2013 A = \frac {33}{29} \times \frac {41}{37} \times \ldots \times \frac {2017}{2013} and B = 37 33 × 45 41 × × 2013 2009 B = \frac {37}{33} \times \frac {45}{41} \times \ldots \times \frac {2013}{2009}

Notice that, by comparing the individual terms, we have 29 25 × B > A \frac {29}{25} \times B > A and 2021 2017 × B < A \frac {2021}{2017} \times B < A

So we have:

B × 2021 2017 < A < 29 25 × B B \times \frac {2021}{2017} < A < \frac {29}{25} \times B

A B × 2021 2017 < A 2 < 29 25 × A B AB \times \frac {2021}{2017} < A^2 < \frac {29}{25} \times AB

2021 29 < A 2 < 2017 25 \frac {2021}{29} < A^2 < \frac {2017}{25}

69 < A 2 < 81 69 < A^2 < 81

8 < A < 9 8 < A < 9

Hence A = 8 \lfloor A \rfloor = \boxed{8}

Kyle Dulay - 7 years, 5 months ago

Log in to reply

I was waiting for an answer like this...thanks Kyle.

I had a solution with the Gamma-function Γ ( x ) \Gamma(x) .

33 29 × 41 37 × × 2017 2013 = \large \frac{33}{29} \times \frac{41}{37} \times \ldots \times \frac{2017}{2013} = ( 2017 8 ) ! / ( 25 8 ) ! ( 2013 8 ) ! / ( 21 8 ) ! = \large \frac{( \frac{2017}{8})! / ( \frac{25}{8})!} {( \frac{2013}{8})! / ( \frac{21}{8})!} = ( 2017 8 ) ! ( 21 8 ) ! ( 2013 8 ) ! ( 25 8 ) ! = \large \frac{( \frac{2017}{8})! ( \frac{21}{8})!} {( \frac{2013}{8})! ( \frac{25}{8})!} =

Γ ( 1 + 2017 8 ) Γ ( 1 + 21 8 ) Γ ( 1 + 2013 8 ) Γ ( 1 + 25 8 ) = 8.63571857... \large \frac{ \Gamma(1+\frac{2017}{8}) \Gamma(1+\frac{21}{8})} {\Gamma(1+\frac{2013}{8}) \Gamma(1+\frac{25}{8})} \normalsize = 8.63571857...

with Γ \Gamma -functions calculated using Mathematica.

Ronald Overwater - 7 years, 5 months ago

Beautifully presented.....awesome!

Eddie The Head - 7 years, 5 months ago

Well said, Kyle. This is basically what I did (except I was less formal).

Peter Byers - 7 years, 5 months ago

How did u managed to get

2021 29 < A 2 < 2017 25 \frac{2021}{29} < A^2 < \frac{2017}{25}

from A B × 2021 2017 < A 2 < 29 25 × A B AB \times \frac{2021}{2017} < A^2 < \frac{29}{25} \times AB ? I know you multiplied both sides by 2017 29 \frac{2017}{29} , but what about the A B AB ?

Fan Zhang - 7 years, 5 months ago

Log in to reply

A B = ( 33 29 × 41 37 × × 2017 2013 ) ( 37 33 × 45 41 × × 2013 2009 ) AB = (\frac {33}{29} \times \frac {41}{37} \times \ldots \times \frac {2017}{2013})(\frac {37}{33} \times \frac {45}{41} \times \ldots \times \frac {2013}{2009})

A B = 33 29 × 37 33 × 41 37 × 45 41 × × 2013 2009 × 2017 2013 AB = \frac {33}{29} \times \frac {37}{33} \times \frac {41}{37} \times \frac {45}{41} \times \ldots \times \frac {2013}{2009} \times \frac {2017}{2013}

A B = 2017 29 AB = \frac {2017}{29}

I didn't multiply 2017 29 \frac {2017}{29} to the leftmost and rightmost sides (which I can't do without also multiplying the middle part by it), but rather I substituted it for A B AB .

Kyle Dulay - 7 years, 5 months ago

We can find upper bound by AM-GM i = 3 251 8 i + 9 8 i + 5 ( i = 3 251 8 i + 9 8 i + 5 ) 249 24 9 249 = 1 2 249 ( 498 + ψ ( 2021 8 ) ψ ( 29 8 ) ) 249 24 9 249 8.88 \prod_{i = 3}^{251} \frac{8i + 9}{8i + 5} \leq \frac{\left(\sum_{i = 3}^{251} \frac{8i + 9}{8i + 5}\right)^{249}}{249^{249}} = \frac{\frac{1}{2^{249}} \cdot \left(498 + \psi\left(\frac{2021}{8}\right) - \psi\left(\frac{29}{8} \right )\right)^{249}}{249^{249}} \approx 8.88 where ψ ( z ) \psi(z) is Digamma function.

Jan J. - 7 years, 5 months ago

Log in to reply

What is that?

A Brilliant Member - 7 years, 5 months ago

lol

Muhammad Shariq - 7 years, 5 months ago

did anybody has manual solution

Ben will - 7 years, 5 months ago

Log in to reply

Check Kyle's......

Eddie The Head - 7 years, 5 months ago

Well,here's a method,but it doesn't give us 8.[ EDIT: Thanks to A Joshi for spotting a mistake.I have corrected it,but this unfortunately gives an worse approximation]

There are 248 expressions in the numerator and 248 in the denominator.All the fractions are of the form x + 4 x \dfrac{x+4}{x} .Since the terms aren't too big but there are way too many terms,we can approximate x + 4 x \dfrac{x+4}{x} as 1.01 1.01 . Therefore,the whole expression becomes

1.01 1.01........1.01 = ( 1.01 ) 248 1.01\cdot 1.01........1.01=(1.01)^{248}

which is approximately 11.79 11.79 .This gives us something close,but not the answer we are looking for.I am sure this method can be improved a bit to give us a closer value.

Rahul Saha - 7 years, 5 months ago

Log in to reply

There are not 24 but 248 expressions !

A Joshi - 7 years, 5 months ago

Log in to reply

Thank you,I have edited my earlier comment and made some suitable changes.Unfortunately,this gives us an worse approximation.

Rahul Saha - 7 years, 5 months ago

Well, considering lim x 0 x + 4 x = \lim_{x \to 0} \frac{x+4}{x} = \infty and lim x x + 4 x = 0 \lim_{x \to \infty} \frac{x+4}{x} = 0 I don't think this is a good way to estimate.

Hao Tang - 7 years, 5 months ago

Log in to reply

Your second limit is incorrect - lim x x + 4 x = lim x 1 + 4 x = 1 \lim_{x \rightarrow \infty} \frac{x+4}{x}=\lim_{x \rightarrow \infty} 1+\frac{4}{x}=1 .

Muhammad Shariq - 7 years, 4 months ago

Kyle, How did you get that 29 25 × B > A \frac {29}{25} \times B > A ? (and the second inequality)

Elliot Gorokhovsky - 7 years, 5 months ago

Log in to reply

We have the ff. inequalities:

29 25 > 33 29 \frac {29}{25} > \frac {33}{29}

37 33 > 41 37 \frac {37}{33} > \frac {41}{37}

until

2013 2009 > 2017 2013 \frac {2013}{2009} > \frac {2017}{2013}

Multiply all the left-hand sides and all the right-hand sides of the inequalities to get:

29 25 × 37 33 × . . . × 2013 2009 > 33 29 × 41 37 × . . . × 2017 2013 \frac {29}{25} \times \frac {37}{33} \times ... \times \frac {2013}{2009} > \frac {33}{29} \times \frac {41}{37} \times ... \times \frac {2017}{2013}

29 25 × B > A \frac {29}{25} \times B > A

The proof for the second inequality is similar.

Kyle Dulay - 7 years, 5 months ago

Log in to reply

Thanks!

Elliot Gorokhovsky - 7 years, 5 months ago

Why did u multiply by A ?

Ravishankar Shukla - 7 years, 5 months ago
Forretrio Wong
Jan 3, 2014

By estimation with the claim 33 29 × 41 47 × . . . × 8 n + 9 8 n + 5 O ( n ) \frac{33}{29}\times \frac{41}{47}\times ... \times \frac{8n+9}{8n+5} \sim O(\sqrt{n}) .

Notice that n + 8 n + 4 n + 4 n = ( n + 4 n ) 2 + O ( n 2 ) \frac{n+8}{n+4}\frac{n+4}{n} = (\frac{n+4}{n})^2 + O(n^{-2}) but can we do better?

Consider n + 8 + k n + k ( n + 4 n ) 2 = ( 16 + 8 k ) n + 16 k n 2 ( n + k ) \frac{n+8+k}{n+k} - (\frac{n+4}{n})^2 = \frac{(16+8k)n+16k}{n^2(n+k)} so that if we set k = 2 k=-2 we get an error term of O ( n 3 ) O(n^{-3}) . When we multiply the error term the degree is increased by 2 since we have n 2 n^2 such error term.

i = 3 n 8 i + 9 8 i + 5 = i = 3 n 8 i + 11 8 i + 3 + O ( i 3 ) \prod_{i=3}^{n} \frac{8i+9}{8i+5} = \prod_{i=3}^{n} \sqrt{\frac{8i+11}{8i+3}}+O(i^{-3})

= i = 3 n ( 8 i + 11 8 i + 3 + O ( i 3 / 2 ) ) = 8 n + 11 27 + O ( n ) = \prod_{i=3}^{n} (\sqrt{\frac{8i+11}{8i+3}}+O(i^{-3/2})) = \sqrt{\frac{8n+11}{27}}+O(\sqrt{n})

So that [ 33 29 × 41 37 × . . . × 8 n + 9 8 n + 5 ] [ 2019 27 ] = 8 [\frac{33}{29} \times \frac{41}{37} \times ... \times \frac{8n+9}{8n+5}]\approx [\sqrt{\frac{2019}{27}}] = \boxed {8}

Please note that it still yield error of order n \sqrt{n} that is very bad for such estimation but it is rather small (about 0.0002603 n \sim 0.0002603 \sqrt{n} ) so it can be neglected for such a small n n .

Should be 8 i + 11 8 i + 3 + O ( i 3 ) \sqrt{\frac{8i+11}{8i+3} + O(i^{-3})} , sorry.

Forretrio Wong - 7 years, 5 months ago

"#include <iostream>

int main()

{

double x = 1;
for(double i = 33; i<=2017; i+=8)

{ x=x*(i/(i-4)); }

cout << x;

system("pause"); return 0;

}

Olexandr Zavalistyi - 7 years, 5 months ago

Log in to reply

haha nice way of doing it

Satya Teja Chikatla - 7 years, 4 months ago

i also did it in the same way

Priyesh Pandey - 7 years, 2 months ago

Can you explain why it is O(sqrt (n)) ??

Eddie The Head - 7 years, 5 months ago

Log in to reply

It's because we have n 2 n^2 such error terms so we multiply n 2 n^2 to the error term inside the product. I used i i inside but this is also related to n n the number of terms so it becomes O ( n ) O(\sqrt{n}) .

If you are talking about the initial claim, it follows from the above calculation that the final result is about a constant times n \sqrt{n} . However, I must admit that I didn't 'prove' that it is really asymptotically equal to O ( n ) O(\sqrt{n}) . To do this, we need an even better estimate, but that would make our calculation inconvenient.

Forretrio Wong - 7 years, 5 months ago

include < iostream > using namespace std;

int main() { double x = 1; for(double i = 33; i<=2017; i+=8) { x=x*(i/(i-4)); } cout « x; system("pause"); return 0; } so answer is [8,63572]=8 :)

Александр Поддубный - 7 years, 5 months ago
Ron van den Burg
Jan 18, 2014

Let A = 37.45...2021 33.41...2017 < B = 33.41...2017 29.37...2013 < C = 29.37...2013 25.33...2009 A=\frac {37.45...2021}{33.41...2017} < B = \frac {33.41...2017}{29.37...2013} < C = \frac {29.37...2013}{25.33...2009}

Thus A . B < B . B < C . B A.B < B.B < C.B . Now A . B = 2021 29 A.B = \frac {2021}{29} and C . B = 2017 25 C.B=\frac {2017}{25} . The result follows from 8 < A . B < B < C . B < 9 8 < \sqrt {A.B} < B < \sqrt {C.B} <9 .

Budi Utomo
Jan 3, 2014

(1,25)^249 = 8, ... So, the answer is 8

WHY 1.25

MNS Muzahid - 7 years, 5 months ago

Ya, how did you get that?

Elliot Gorokhovsky - 7 years, 5 months ago

ok.. i use assumtion that (33/29)...(2017/2013) = 1,12 ...(249)...1,001 = | (1,25)^249 | = 8. [ n = 249]

Budi Utomo - 7 years, 5 months ago

Log in to reply

(1.25)^249 = 1.35x10^24....so not between 8 and 9!

(1.00839)^249= 8.008, (1.00886)^249 = 8.993

Ronald Overwater - 7 years, 4 months ago

India me esa nahi chalta hai

Purvam Modi - 7 years, 3 months ago

If we recast the product in the form : (1+4/29)(1+4/37)...the natural log could be expanded in taylor series for ln(1+x) which can be approximated by the first term x resulting in a series of the form 4/29 + 4/37 +......248 terms. . I was also wondering if we could approximate the sum involved by an integral of the form : 4*$ dx / (29+8x) with x ranging from 1 to 248. The difference between the above integral and the discrete sum would most possibly be less than 1 , in fact it may be even lesser than euler's constant which is approximately 0.5

Sundar R - 7 years, 3 months ago

0 pending reports

×

Problem Loading...

Note Loading...

Set Loading...